Ehh INMO was trash. I think I will get 17/0/0/0-1/3-5/10-14, which is def not good enough for qualifying from 12th grade. Well, I really feel sad but let's not talk about it and focus on EGMO rather.
INMO 2023 P1
Let S be a finite set of positive integers. Assume that there are precisely 2023 ordered pairs (x,y) in S\times S so that the product xy is a perfect square. Prove that one can find at least four distinct elements in S so that none of their pairwise products is a perfect square.
I will use Atul's sol, cause it's the exact same as mine.
Proof: Consider the graph G induced by the elements of S and edges being if the products are perfect squares. Note that if xy = a^2 and xz = b^2, then yz = \left( \frac{ab}{x} \right)^2, since its an integer and square of a rational number its a perfect square and so yz is an edge too. So the graph is a bunch of disjoint cliques, say with sizes c_1, c_2, \cdots, c_k. Then \sum_{i=1}^k c_i^2 = 2023, which implies k \geqslant 4 since 2023 \equiv 7 \pmod 8. Now pick one element from each of these cliques and it works.
I am starting with Harmonic DGX:
A few stuffs to note for me.
2016 G2
Let ABC be a triangle with circumcircle \Gamma and incenter I and let M be the midpoint of \overline{BC}. The points D, E, F are selected on sides \overline{BC}, \overline{CA}, \overline{AB} such that \overline{ID} \perp \overline{BC}, \overline{IE}\perp \overline{AI}, and \overline{IF}\perp \overline{AI}. Suppose that the circumcircle of \triangle AEF intersects \Gamma at a point X other than A. Prove that lines XD and AM meet on \Gamma.
Proof: Define T as the mixtillinear in touch point. Define S, P as the midpoint of major arc, minor arc BC.
Claim: BFIT, CEIT harmonic
Proof: Introduce the midpoint of arc AB, BC ( say M_c,M_b) and angle chase. We have M_cB,M_cI tangent to BFIT. And we have T-F-M_c.
Define H=ST\cap BC.
Claim: AH,CF,BE concur
Proof: By ceva, we just have to show \frac{BF}{CE}=\frac{BH}{CH}.
For which, we use the fact that BFIT, CEIT are harmonic.
Claim: SX,EF,BC,TP concur
Proof: Note that (Z,H;B,C)=-1 where Z=EF\cap BC. Moreover, we know that \angle BTH=HTC
and
\angle HTP=90\implies Z-T-P.
As SH\perp ZP, ZH\perp PS\implies X-H-P.
Since (S,P;B,C)=-1, we get (X,T;B,C)=-1.
Now define A' as the intersection of the parallel line through A to BC and (ABC).
Claim: T-D-A'
Proof: We have it because \angle BTA=CTD
and
BA=CA'.
Note that (AM\cap (ABC),A';B,C)=-1 ( just reflect over perpendicular bisector of BC) . But we have (X,T;B,C)=(XD\cap (ABC),A';B,C)=-1. So XD\cap (ABC)=AM\cap (ABC).
This was a pretty nice problem (sort of config based).. but I would recommend to go through it in future, cause pretty nice config geo!
USA EGMO TST 2023 P1
Let ABC be a triangle with AB+AC=3BC. The B-excircle touches side AC and line BC at E and D, respectively. The C-excircle touches side AB at F. Let lines CF and DE meet at P. Prove that \angle PBC = 90^{\circ}.
Proof: If BC=a. Note that CE=CD=\frac{AB+BC+CA}{2}-C=a. So C is the centre of (BDE) and hence \angle BDE=90. So C is midpoint of BE.
Define I as the incentre of ABC. Let the incircle touch CA at M, BC at L. Let M antipode be N and L antipode be J.
Claim:B-N-J-D collinear ( By symmetry we also get C-J-F)
Proof: By homothety, B-N-D. Note that if \angle BCA=2\theta, we get \angle BED=\theta\implies CI||EA.
We have \angle EBD=90-\theta and \angle ILM =\theta\implies \Delta LMJ \text{ is similar to } \Delta EDB.
Note that \angle MJN=90. Since JL\perp BC\implies \angle LJB=90-\angle LBJ=\theta. And \angle LJN=\angle JLM=\theta\implies J-N-B.
Define P' as the intersection of the perpendicular at B to BC and CE. Define C'=CI\cap BC. Since LJ||BP' and I is midpoint of NJ\implies C' is midpoint of BP'.
Now define Y as midpoint of NJ. Note that C-I-Y-C' ( as CI is perpendicular bisector of LM and LMNJ rectangle).
Note that \Delta BCY is similar to \Delta BED. So E-D-P'. So P'=P. And we are done!
USAJMO 2011 P5
Points A,B,C,D,E lie on a circle \omega and point P lies outside the circle.
The given points are such that
(i) lines PB and PD are tangent to \omega,
(ii) P, A, C are collinear,
and (iii) {DE} \parallel {AC}.
Prove that {BE} bisects {AC}.
Proof: Note that
(A,C;B,D)=-1\text { projecting through } E \text { on } AC\implies (A,C; BE\cap AC, P_{\infty})=-1.
Hence
BE\cap AC is the midpoint of
AC. Hence
BE bisects
AC.
APMO 2013
Let ABCD be a quadrilateral inscribed in a circle \omega, and let P be a point on the extension of {AC} such that {PB} and {PD} are tangent to \omega. The tangent at C intersects {PD} at Q and the line AD at R. Let E be the second point of intersection between {AQ} and \omega. Prove that B, E, R are collinear.
Proof: Note that
-1=(A,E;C,D)=(A,RE\cap \omega; C,D)
But we know
(A,C;B,D)=-1\implies RE\cap \omega=B.
China TST 2002
Let ABCD be a quadrilateral. Point E is the intersection of lines AB and CD
while point F is the intersection of lines BC and DA. The diagonals of the quadrilateral meet at P, and point O is the foot from P to \overline{EF}.
Prove that \angle BOC = \angle AOD.
Proof: Let
DB\cap FE=X, FP\cap DC=Y. Note that
-1=(D,C;Y,E)=(D,B;P,Y)\implies PO\text{ bisects } \angle DOB.
Similarly we get
PO \text{ bisects } \angle AOC. Hence
\angle BOC = \angle AOD.
ELMO Shortlist 2019 G1
Let ABC be an acute triangle with orthocenter H and circumcircle \Gamma.
Let BH intersect AC at E, and let CH intersect AB at F. Let AH intersect \Gamma again at P \neq A. Let PE intersect \Gamma again at Q \neq P.
Prove that BQ bisects segment \overline{EF}.
Proof: Let the parallel line through B to EF intersect (ABC) at X. Let EF\cap BC=T, TA\cap (ABC)=R. Then note that (AREF), R-E-X.
So
-1=(B,C;T,D)\text{ projecting through } A \text { on } (ABC)
\implies -1= (BC; RP)\text{ projecting through } A \text { on } (ABC)\text{ projecting through } E \text { on } (ABC)
\implies (BE\cap (ABC)A;XQ)=-1\text{ projecting through } E \text { on } (ABC)\implies (EF;\infty_{EF}BQ\cap EF)=-1.
We have
FM\cdot FX=MA\cdot MD=MY\cdot MF\implies EYXF\text{ is cyclic}.
The following problem was done with Atul and Archit :D
SAGF part 1 P2
In an acute-angled triangle, ABC w is a circumscribed circle, and O is inside the triangle and OB = OC. The point D is selected on the side of AB,so that OD \parallel BC. The straight line AO repeatedly intersects the circles w and (COD) at the points A',P, respectively. M is the middle of the side of BC. Circle (MAA') intersects the line BC at the point L. The straight line AL repeatedly intersects the circle w at the point K. Prove that \angle APK = \angle AA'M
Proof: We start with the following lemma.
Lemma:Let ABC be a triangle, O be a point on the perpendicular bisector of BC. Let D,E denote the intersection of line parallel AB,AC. Define P=(DOC)\cap (EOP). Then points A,O and P are collinear.
Proof: Invert about O. The inverted problem is: Give ABC triangle with O be a point on the perpendicular bisector of BC. Let D,E denote the intersection of line parallel (AOB),(AOC). We have to show that AO, CD,BE concur.
Note that as
OB=OC we have
\angle DAB=\angle DOB=\angle EOC=\angle EAC.
Define
\ell_1 as the reflection of
DB over the angle bisector of
\angle B and
\ell_2 as the reflection of
CE over the angle bisector of
\angle C. We claim that
\ell_1 \parallel \ell_2 \parallel OP
To see this,
\angle AOD=\theta=\angle ABD\implies \angle AOE=180-\theta=\angle ACE.
Let
X be on
\ell_1 such that
D and
X are on opposite sides of
AB. Similarly, define
Y be on
\ell_2 such that
E and
Y are on opposite sides of
AC. So note that
\angle CBX=\theta,\angle BCY=180-\theta\implies \ell_1 \parallel \ell_2.
Note that
\angle ABX=\angle B+\theta. And
180-\angle OAB-180-\angle ODB=\angle DBC=\angle B+\theta\implies OA||l_1||l_2.
So
BX\cap CY=P_{\infty}, A-O-P_{\infty}\text{ collinear}. By Jacobi theorem, we get
A-O-P_{\infty},CD,BE concur. Hence, we get
A-O-P collinear.
Redefine the problem in the following way: Define trinagle
ABC,
O be a point on the perpendicular bisector of
BC. Let
D,F denote the intersection of line parallel
AB,AC. Define
P=(DOC)\cap (EOP). Define
M as the midpoint of
BC. Define
I=AB\cap (DOCP), H=AC\cap (BOFP). Define
L'= BC\cap IH. Define
E=AO\cap BC, A'=AO\cap (ABC). We will shwo that
L',A',M,A cyclic i.e
L'=L. Note that
(L',E;B,C)=-1. And hence we have
ME\cdot EL'=EC\cdot EB=AE\cdot EA'\implies L'MAA'\text{ is cyclic}\implies L'=L.
Due to this, we get that K is the miquel point of BCHI(as K=(ABC)\cap AL). Since K is the miquel point, we have (ABCK),(AIHK),(KCHL) cyclic. We actually don't need all the cyclicities, but we'll prove them because why not.
Note that (B,C;E,L)=-1\implies LB\cdot LC=LM\cdot LE. But LB\cdot LC=LA\cdot LK\implies AKEM is cyclic.
P\in (AKHI) as
\angle IPH=\angle IPO+\angle OPH=\angle ADO+\angle AFO=180-\angle A\implies P\in(AKI).
Define
G=DF\cap AL, then
KFGH cyclic, this is because
\angle KGF=\angle KLC=\angle KLB=\angle KIB=\angle KHC.
The last equality follows as
K is the spiral center taking
BI to
CH.
So by PoP, we get
AK\cdot AG=AF\cdot AH=AO\cdot AP=AD\cdot AI
which gives
KDIG,KGOP cyclic.
EKLP cyclic as
\angle KLE=\angle KLB=\angle KIB=\angle KIA=\angle KPA=\angle KPE.
AKME cyclic as
LM\cdot LE=LB\cdot LC=LA\cdot LK\implies AKME\text{ is cyclic}.
Now to finish proving
\angle APK=\angle AA'M, note that
\angle APK=\angle AIK=\angle BIK=\angle BLK=\angle MLA=\angle AA'M
so we're done.
GOTEEM 2020
Let ABC be a scalene triangle. The incircle of \triangle ABC is tangent to sides {BC}, {CA}, {AB} at D, E, F, respectively. Let G be a point on the incircle of \triangle ABC such that \angle AGD = 90^\circ. If lines DG and EF intersect at P,prove that AP is parallel to BC.
Proof: Let
D' be the antipode of
D wrt incircle. Note that
(D',G;E,F)=-1\text { projecting through } D \text { on } EF\implies (DD'\cap EF, P;E,F)=-1.
Let
M be the midpoint of
BC. Define
X=EF\cap BC. We have
EF,DD',AM concur at say
R. Projecting through
R on
BC,
-1=(R,P;E,F)=(M,PA\cap BC;B,C)\implies PA||BC.
AHH I have more to post but I will post in different blogpost! Bie people!
Comments
Post a Comment